If integer \(k\) is equal to the sum of all even multiples of \(15\) between \(295\) and \(615,\) what is the greatest

This topic has expert replies
Legendary Member
Posts: 2276
Joined: Sat Oct 14, 2017 6:10 am
Followed by:3 members

Timer

00:00

Your Answer

A

B

C

D

E

Global Stats

If integer \(k\) is equal to the sum of all even multiples of \(15\) between \(295\) and \(615,\) what is the greatest prime factor of \(k?\)

A. 5
B. 7
C. 11
D. 13
E. 17

Answer: C

Source: Manhattan GMAT

GMAT/MBA Expert

User avatar
GMAT Instructor
Posts: 16207
Joined: Mon Dec 08, 2008 6:26 pm
Location: Vancouver, BC
Thanked: 5254 times
Followed by:1268 members
GMAT Score:770
VJesus12 wrote:
Thu May 13, 2021 6:51 am
If integer \(k\) is equal to the sum of all even multiples of \(15\) between \(295\) and \(615,\) what is the greatest prime factor of \(k?\)

A. 5
B. 7
C. 11
D. 13
E. 17

Answer: C

Source: Manhattan GMAT
Multiples of 15: 15, 30, 45, 60, 75, 90, 105, etc
EVEN multiples of 15: 30, 60, 90, 120, ....

So k = 300 + 330 + 360 + ... + 570 + 600
Since each number is a multiple of 30, let's rewrite each value as the product of 30 and some integer:
300 = 30(10)
330 = 30(11)
360 = 30(12)
390 = 30(13)
.
.
.
570 = 30(19)
600 = 30(20)

So k = 30(10 + 11 + 12 + ... + 19 + 20)

------------------------------------------------------
Let's examine this sum: 10 + 11 + 12 + ... + 19 + 20
Since 20 - 10 + 1 = 11, we know there are 11 numbers to add together.

Since these red numbers are equally spaced (consecutive integers), their sum = (# of values)(average of first and last values)
= [11][(10+20)/2]
= [11][15]
= (11)(15)

-------------------------------------------------
So, k = 30(10 + 11 + 12 + ... + 19 + 20)
= 30(11)(15)
= (2)(3)(5)(11)(3)(5)

We can see that 11 is the greatest prime factor of k

Answer:C

Cheers,
Brent
Brent Hanneson - Creator of GMATPrepNow.com
Image